Wave in inhomogeneous medium (ray equation)

In summary, the problem involves finding the angle ##\theta## between a ray and the y-axis in a stratified medium with varying index of refraction. Using Snell's Law and the chain rule, it is possible to show that ##\frac{d\theta}{ds} = \frac{-(dn/dy) sin(\theta)}{n}##, where s is measured along the ray. This result can be used to verify the ray equation ##\frac{d}{ds} (nt) = \nabla n##, where t is a unit vector tangent to the ray at a point where the index of refraction is n. The product rule and the relation ##cos(\theta) = \frac
  • #1
deedsy
81
0

Homework Statement


A wave travels in a stratified medium whose index of refraction is a function of the coordinate y. Show that the angle ##\theta## between a ray and the y-axis obeys the following law:
## \frac{d\theta}{ds} = \frac{-(dn/dy) sin(\theta)}{n} ## , where the distance s is measured along the ray.

Using this result, you can verify the ray equation ##\frac{d}{ds} (nt) = \nabla n## , where t is a unit vector tangent to the ray at a point where the index of refraction is n.

Hint: Select your y-axis along ##\nabla n## and your x-axis in the plane of incidence.

Homework Equations


Snells Law --> ##n_1 sin(\theta_1) = n_2 sin(\theta_2)##

The Attempt at a Solution


I need an equation that relates ##\theta, s, y, ##and ##n(y) ##, which shouldn't be hard but I can't seem to figure it out.

I attached a simple figure of how I am setting this up.
From the figure, s and y are related by
## cos(\theta) = \frac{dy}{ds} ##
##\theta = \cos^{-1}(\frac{dy}{ds})##

n and ##\theta## are related by Snell's Law, which for this problem, can be applied to a large number of thin y-axis layers with varying refractive indexes
## n_1 sin(\theta_1) = n_2 sin(\theta_2) = n_3 sin(\theta_3) = A##, where A is just a constant
##n = \frac{A}{\sin(\theta)}##
## \frac{dn}{d\theta} = \frac{-A cos(\theta)}{sin^2(\theta)}##

With these expressions for ## dn, dy, ds, d\theta##, I've just been manipulating these expressions trying to match what the answer should be, but I'm not making any progress. Am I on the right track trying to solve this using only the geometry of the problem and Snell's law, or is there another way someone can reccomend? I've seen the ray equation derived by using phase differences and fermat's principle, but those didn't yield what the first part of this question is asking for.
 

Attachments

  • Screen Shot 2015-04-17 at 11.17.11 AM.png
    Screen Shot 2015-04-17 at 11.17.11 AM.png
    14.9 KB · Views: 456
Physics news on Phys.org
  • #2
deedsy said:
## \frac{dn}{d\theta} = \frac{-A cos(\theta)}{sin^2(\theta)}##

Try invoking the chain rule to re-express ## \frac{dn}{d\theta}##.
 
  • Like
Likes deedsy
  • #3
thanks TSny, that tip led me to the right answer!

Now I'm trying to verify the ray equation ##\frac{d}{ds} (nt) = \nabla n## with that result; so far,
##\frac{d\theta}{ds} = \frac{-(dn/dy) sin(\theta)}{n}##
since n is only dependent on the y direction, ##\nabla n = \frac{dn}{dy}##, so
##\frac{-d\theta}{ds} \frac{n}{sin(\theta)} = \nabla n##
I then tried taking ##\frac{d}{ds}## of both sides, but couldn't reduce it to the ray equation

so, somehow ##\frac{-d\theta}{ds} \frac{n}{sin(\theta)}## must equal ##\frac{d}{ds} (nt)## , where t is "a unit vector tangent to the ray at a point where the index of refraction is n".
 
  • #4
deedsy said:
##\frac{-d\theta}{ds} \frac{n}{sin(\theta)} = \nabla n##
This looks good assuming that here ##\nabla n## refers to the magnitude of the gradient: ##|\vec{\nabla} n|##.

You can use this result to show that ##\frac{d}{ds} \left ( n \hat{t} \right )## reduces to ## \vec{\nabla} n##, where here ## \vec{\nabla} n## is the gradient vector (not the magnitude).

I doubt if I have the most elegant way to do it, but I found it helpful to express the unit tangent vector ##\hat{t}## in terms of Cartesian unit vectors ##\hat{i}## and ##\hat{j}## and the angle ##\theta## between ##\hat{t}## and the y-axis.

[EDIT: Actually, it also works out nicely if you don't bother to express ##\hat{t}## in terms of ##\hat{i}## and ##\hat{j}##. Either way, my hint would be to use the product rule on ##\frac{d}{ds} \left ( n \hat{t} \right )## and see if you can simplify it.]
 
Last edited:
  • #5
Okay, so I'm going to reduce ##\frac{d}{ds} (n \hat{t})## down to ##\vec{\nabla}n##
##\frac{d}{ds} (n \hat{t})##
##\frac{dn}{ds} + \frac{d\hat{t}}{ds}##
Using a previous result...
##\frac{-A cos(\theta)}{sin^2(\theta)} \frac{d\theta}{ds} + \frac{d\hat{t}}{ds}##
Using another previous result...
##\frac{A cos(\theta)}{sin^2(\theta)} \frac{\nabla n sin(\theta)}{n} + \frac{d\hat{t}}{ds}##
##\frac{A cos(\theta) \nabla n}{n sin(\theta)} + \frac{d\hat{t}}{ds}##
##(\nabla n)(cos(\theta)) + \frac{d\hat{t}}{ds}##

Now, since ##\hat{t}## is always tangent to ##ds##, I believe ##\frac{d\hat{t}}{ds} = 0##

There is also the relation ##cos(\theta) = \frac{dy}{\hat{t}}##

So,##(\nabla n)(cos(\theta)) + \frac{d\hat{t}}{ds} = \nabla n \frac{dy}{\hat{t}} = \vec{\nabla}n##?
I'm not sure about the last step there, though..
 
  • #6
deedsy said:
Okay, so I'm going to reduce ##\frac{d}{ds} (n \hat{t})## down to ##\vec{\nabla}n##
##\frac{d}{ds} (n \hat{t})##
##\frac{dn}{ds} + \frac{d\hat{t}}{ds}##
This isn't correct. The product rule for derivatives gives ##\frac{d}{ds} (n \hat{t}) = \frac{dn}{ds}\hat{t} + n \frac{d\hat{t}}{ds} ##

But you are right that ##\frac{dn}{ds} = |\vec{\nabla} n| \cos \theta##. Another way to get this is to write ##\frac{dn}{ds} = \frac{dn}{dy} \frac{dy}{ds}## and note that ## \frac{dn}{dy} = |\vec{\nabla} n|## and ##\frac{dy}{ds} = \cos \theta##
Now, since ##\hat{t}## is always tangent to ##ds##, I believe ##\frac{d\hat{t}}{ds} = 0##
This is not zero. As you move along the ray, the unit tangent vector will change direction.

There is also the relation ##cos(\theta) = \frac{dy}{\hat{t}}##
You can't divide by a vector.
 
Last edited:
  • #7
wow, yeah, so after the real product rule, I have
##\frac{d}{ds} (n\hat{t}) = \nabla n cos(\theta) \hat{t} + n \frac{d\hat{t}}{ds}##

so I need an expression that can show how ##\hat{t}## changes with ##s##, but I'm having trouble with that.
You said before you found an expression between ##\theta##, ##\hat{t}## and the y-axis; is this the expression that can be useful for this part? I don't see what else this could be besides ##cos(\theta) \hat{t} = dy##, but that doesn't work because you can't divide by a unit vector.
 
  • #8
deedsy said:
wow, yeah, so after the real product rule, I have
##\frac{d}{ds} (n\hat{t}) = \nabla n cos(\theta) \hat{t} + n \frac{d\hat{t}}{ds}##
OK
so I need an expression that can show how ##\hat{t}## changes with ##s##, but I'm having trouble with that.
Try the chain rule again. ##\frac{d\hat{t}}{ds} = \frac{d\hat{t}}{d \theta} \frac{d\theta}{ds}##

You said before you found an expression between ##\theta##, ##\hat{t}## and the y-axis; is this the expression that can be used for this part?

Yes. How would you express the unit tangent vector ##\hat{t}## in terms of ##\hat{i}##, ##\hat{j}##, and ##\theta##?
 
  • #9
##\frac{d\hat{t}}{ds} = \frac{d\hat{t}}{d\theta} \frac{d\theta}{ds}##
##\frac{d\hat{t}}{d\theta} \frac{d\theta}{ds} = \frac{-d\hat{t}}{d\theta} \frac{\nabla n sin(\theta)}{n}##
##\frac{d\hat{t}}{ds} = -\frac{\nabla n sin(\theta)}{n} \frac{d\hat{t}}{d\theta}##

so now I need my expression relating ##\theta## and ##\hat{t}##

##\hat{t} = (ds) sin(\theta) \hat{i} + (ds) cos(\theta) \hat{j}##
##\frac{d\hat{t}}{d\theta} = ds cos(\theta) \hat{i} - ds sin(\theta) \hat{j}##

##\frac{d\hat{t}}{ds} = -\frac{\nabla n sin(\theta)}{n} [ds cos(\theta) \hat{i} - ds sin(\theta) \hat{j}]##

Does this look right so far? If so, next I'll plug it into
##\frac{d}{ds} (n\hat{t}) = \nabla n cos(\theta) \hat{t} + n \frac{d\hat{t}}{ds}##
 
Last edited:
  • #10
actually, I'm not sure i need the (ds) in the unit vector equation
 
  • #11
ok, without including the (ds) I was able to get the answer!
##\frac{d}{ds} (n\hat{t}) = \nabla n\hat{j}##

Thanks so much for your help TSny
 
  • #12
deedsy said:
ok, without including the (ds) I was able to get the answer!
##\frac{d}{ds} (n\hat{t}) = \nabla n\hat{j}##
That looks right. Good work! (Yes, the ds should not be in the expression for the unit tangent vector.)
 

1. What is a "wave in inhomogeneous medium"?

A wave in inhomogeneous medium refers to a type of wave that propagates through a medium that has varying properties, such as density, temperature, or refractive index. This causes the wave to change direction or speed as it travels through the medium, leading to complex behaviors.

2. What is the ray equation?

The ray equation is a mathematical expression that describes the propagation of a wave through an inhomogeneous medium. It takes into account the varying properties of the medium and allows for the calculation of the direction and speed of the wave at different points in the medium.

3. How is the ray equation derived?

The ray equation is derived from the wave equation, which describes the behavior of waves in homogeneous media. By incorporating the varying properties of an inhomogeneous medium into the wave equation, the ray equation is obtained. This is typically done using techniques from calculus and differential equations.

4. What are some applications of the ray equation?

The ray equation has a wide range of applications in various fields such as geophysics, optics, and acoustics. It is used to model the propagation of seismic waves through the Earth's crust, the behavior of light in non-uniform media, and the transmission of sound in the ocean, among others.

5. What are the limitations of the ray equation?

While the ray equation is a useful tool for analyzing wave propagation in inhomogeneous media, it has some limitations. It does not account for wave phenomena such as diffraction or interference, which can occur in certain situations. Additionally, the ray equation assumes that the medium is continuous and smooth, which may not always be the case in real-world scenarios.

Similar threads

  • Advanced Physics Homework Help
Replies
4
Views
457
  • Advanced Physics Homework Help
Replies
3
Views
950
  • Advanced Physics Homework Help
Replies
1
Views
630
  • Advanced Physics Homework Help
Replies
19
Views
841
Replies
1
Views
781
  • Advanced Physics Homework Help
Replies
7
Views
2K
  • Advanced Physics Homework Help
Replies
7
Views
1K
  • Advanced Physics Homework Help
Replies
1
Views
965
  • Advanced Physics Homework Help
Replies
4
Views
1K
  • Advanced Physics Homework Help
Replies
1
Views
669
Back
Top